$Pr[sum_i X_i^2 Y_i^2ge t]$, Chernoff bound for sum of pairs of squared Normal random variables












3












$begingroup$


I'm interested in finding tail bound for $sum_{i=1}^k X_i^2 Y_i^2$, where $X_i$ and $Y_i$ are independent standard normal random variables.
It should be roughly as tight as the standard Chernoff bound, something like $e^{-Omega(ksqrt t)}$ would be nice.



My first instinct was to look at the mgf. $exp(t X^2 Y^2)$, but naturally it doesn't exist. I looked at $exp(i t X^2 Y^2) = e^{i/(8t)} K_0(i/(8t))/sqrt{pi i t}$, but I don't know how to get a tail bound using the characteristic function. I also considered moment bounds. We have $E(X^2 Y^2)^k=2^{2k}Gamma(k+1/2)^2/pile2(2k/e)^{2k}$, but to get a tail bound I need $E(sum_i X_i^2 Y_i^2)^k$, which is of course a lot harder to estimate. I also considered the Cauchy Schwarz bound: $E(sum_i X_i^2 Y_i^2)^kle E(sum_i X_i^4)^{k/2}(sum_i Y_i^4)^{k/2}=E(sum_i X_i^4)^{k}$, but even those moments seem pretty involved.



We have that $Pr[sum_{i=1}^kX_iY_ige tk]le expleft(frac{-t^2k}{2+t}right)$ by Chernoff bounds, which is close to gaussian at least for small $t$. Perhaps we might also expect that $sum X^2_iY^2_i$ is close to Chi-Squared for small $t$?



Does anyone know if there is a standard bound for this sum? Or if one of my approaches might be workable?










share|cite|improve this question











$endgroup$












  • $begingroup$
    Hm, I'm not sure you can do much better than the Markov inequality bound on high moments. $X$ is a sub-gaussian variable, so $X^2$ is a sub-exponential, which means $X^2Y^2$ is worse than that. As you said, the mgf doesn't exist so your best bet is to find bounds on high moments. It's probably unavoidable to expand things out and wade into the combinatorics of gaussian moments.
    $endgroup$
    – zoidberg
    Dec 12 '18 at 2:04










  • $begingroup$
    Actually, take a look at this: math.stackexchange.com/questions/1652781/…
    $endgroup$
    – zoidberg
    Dec 12 '18 at 2:13










  • $begingroup$
    So I think the bound you'll get will be $e^{-Omega(sqrt{t})}$
    $endgroup$
    – zoidberg
    Dec 12 '18 at 2:56










  • $begingroup$
    I think you are right. It seems that $frac{1}{b}sum_{i=1}^kX_i^2Y_i^2>(frac{1}{b}sum_{i=1}^kX_i^2)^2$ close to 50% of the time. So the values seems very similar. Using the later value we get $expleft(frac{-k}{4} left(2 sqrt{t}-2-log (t)right)right)$, which would be great.
    $endgroup$
    – Thomas Ahle
    Dec 12 '18 at 15:56


















3












$begingroup$


I'm interested in finding tail bound for $sum_{i=1}^k X_i^2 Y_i^2$, where $X_i$ and $Y_i$ are independent standard normal random variables.
It should be roughly as tight as the standard Chernoff bound, something like $e^{-Omega(ksqrt t)}$ would be nice.



My first instinct was to look at the mgf. $exp(t X^2 Y^2)$, but naturally it doesn't exist. I looked at $exp(i t X^2 Y^2) = e^{i/(8t)} K_0(i/(8t))/sqrt{pi i t}$, but I don't know how to get a tail bound using the characteristic function. I also considered moment bounds. We have $E(X^2 Y^2)^k=2^{2k}Gamma(k+1/2)^2/pile2(2k/e)^{2k}$, but to get a tail bound I need $E(sum_i X_i^2 Y_i^2)^k$, which is of course a lot harder to estimate. I also considered the Cauchy Schwarz bound: $E(sum_i X_i^2 Y_i^2)^kle E(sum_i X_i^4)^{k/2}(sum_i Y_i^4)^{k/2}=E(sum_i X_i^4)^{k}$, but even those moments seem pretty involved.



We have that $Pr[sum_{i=1}^kX_iY_ige tk]le expleft(frac{-t^2k}{2+t}right)$ by Chernoff bounds, which is close to gaussian at least for small $t$. Perhaps we might also expect that $sum X^2_iY^2_i$ is close to Chi-Squared for small $t$?



Does anyone know if there is a standard bound for this sum? Or if one of my approaches might be workable?










share|cite|improve this question











$endgroup$












  • $begingroup$
    Hm, I'm not sure you can do much better than the Markov inequality bound on high moments. $X$ is a sub-gaussian variable, so $X^2$ is a sub-exponential, which means $X^2Y^2$ is worse than that. As you said, the mgf doesn't exist so your best bet is to find bounds on high moments. It's probably unavoidable to expand things out and wade into the combinatorics of gaussian moments.
    $endgroup$
    – zoidberg
    Dec 12 '18 at 2:04










  • $begingroup$
    Actually, take a look at this: math.stackexchange.com/questions/1652781/…
    $endgroup$
    – zoidberg
    Dec 12 '18 at 2:13










  • $begingroup$
    So I think the bound you'll get will be $e^{-Omega(sqrt{t})}$
    $endgroup$
    – zoidberg
    Dec 12 '18 at 2:56










  • $begingroup$
    I think you are right. It seems that $frac{1}{b}sum_{i=1}^kX_i^2Y_i^2>(frac{1}{b}sum_{i=1}^kX_i^2)^2$ close to 50% of the time. So the values seems very similar. Using the later value we get $expleft(frac{-k}{4} left(2 sqrt{t}-2-log (t)right)right)$, which would be great.
    $endgroup$
    – Thomas Ahle
    Dec 12 '18 at 15:56
















3












3








3





$begingroup$


I'm interested in finding tail bound for $sum_{i=1}^k X_i^2 Y_i^2$, where $X_i$ and $Y_i$ are independent standard normal random variables.
It should be roughly as tight as the standard Chernoff bound, something like $e^{-Omega(ksqrt t)}$ would be nice.



My first instinct was to look at the mgf. $exp(t X^2 Y^2)$, but naturally it doesn't exist. I looked at $exp(i t X^2 Y^2) = e^{i/(8t)} K_0(i/(8t))/sqrt{pi i t}$, but I don't know how to get a tail bound using the characteristic function. I also considered moment bounds. We have $E(X^2 Y^2)^k=2^{2k}Gamma(k+1/2)^2/pile2(2k/e)^{2k}$, but to get a tail bound I need $E(sum_i X_i^2 Y_i^2)^k$, which is of course a lot harder to estimate. I also considered the Cauchy Schwarz bound: $E(sum_i X_i^2 Y_i^2)^kle E(sum_i X_i^4)^{k/2}(sum_i Y_i^4)^{k/2}=E(sum_i X_i^4)^{k}$, but even those moments seem pretty involved.



We have that $Pr[sum_{i=1}^kX_iY_ige tk]le expleft(frac{-t^2k}{2+t}right)$ by Chernoff bounds, which is close to gaussian at least for small $t$. Perhaps we might also expect that $sum X^2_iY^2_i$ is close to Chi-Squared for small $t$?



Does anyone know if there is a standard bound for this sum? Or if one of my approaches might be workable?










share|cite|improve this question











$endgroup$




I'm interested in finding tail bound for $sum_{i=1}^k X_i^2 Y_i^2$, where $X_i$ and $Y_i$ are independent standard normal random variables.
It should be roughly as tight as the standard Chernoff bound, something like $e^{-Omega(ksqrt t)}$ would be nice.



My first instinct was to look at the mgf. $exp(t X^2 Y^2)$, but naturally it doesn't exist. I looked at $exp(i t X^2 Y^2) = e^{i/(8t)} K_0(i/(8t))/sqrt{pi i t}$, but I don't know how to get a tail bound using the characteristic function. I also considered moment bounds. We have $E(X^2 Y^2)^k=2^{2k}Gamma(k+1/2)^2/pile2(2k/e)^{2k}$, but to get a tail bound I need $E(sum_i X_i^2 Y_i^2)^k$, which is of course a lot harder to estimate. I also considered the Cauchy Schwarz bound: $E(sum_i X_i^2 Y_i^2)^kle E(sum_i X_i^4)^{k/2}(sum_i Y_i^4)^{k/2}=E(sum_i X_i^4)^{k}$, but even those moments seem pretty involved.



We have that $Pr[sum_{i=1}^kX_iY_ige tk]le expleft(frac{-t^2k}{2+t}right)$ by Chernoff bounds, which is close to gaussian at least for small $t$. Perhaps we might also expect that $sum X^2_iY^2_i$ is close to Chi-Squared for small $t$?



Does anyone know if there is a standard bound for this sum? Or if one of my approaches might be workable?







probability probability-distributions normal-distribution distribution-tails






share|cite|improve this question















share|cite|improve this question













share|cite|improve this question




share|cite|improve this question








edited Dec 12 '18 at 16:48







Thomas Ahle

















asked Dec 11 '18 at 11:26









Thomas AhleThomas Ahle

1,5121320




1,5121320












  • $begingroup$
    Hm, I'm not sure you can do much better than the Markov inequality bound on high moments. $X$ is a sub-gaussian variable, so $X^2$ is a sub-exponential, which means $X^2Y^2$ is worse than that. As you said, the mgf doesn't exist so your best bet is to find bounds on high moments. It's probably unavoidable to expand things out and wade into the combinatorics of gaussian moments.
    $endgroup$
    – zoidberg
    Dec 12 '18 at 2:04










  • $begingroup$
    Actually, take a look at this: math.stackexchange.com/questions/1652781/…
    $endgroup$
    – zoidberg
    Dec 12 '18 at 2:13










  • $begingroup$
    So I think the bound you'll get will be $e^{-Omega(sqrt{t})}$
    $endgroup$
    – zoidberg
    Dec 12 '18 at 2:56










  • $begingroup$
    I think you are right. It seems that $frac{1}{b}sum_{i=1}^kX_i^2Y_i^2>(frac{1}{b}sum_{i=1}^kX_i^2)^2$ close to 50% of the time. So the values seems very similar. Using the later value we get $expleft(frac{-k}{4} left(2 sqrt{t}-2-log (t)right)right)$, which would be great.
    $endgroup$
    – Thomas Ahle
    Dec 12 '18 at 15:56




















  • $begingroup$
    Hm, I'm not sure you can do much better than the Markov inequality bound on high moments. $X$ is a sub-gaussian variable, so $X^2$ is a sub-exponential, which means $X^2Y^2$ is worse than that. As you said, the mgf doesn't exist so your best bet is to find bounds on high moments. It's probably unavoidable to expand things out and wade into the combinatorics of gaussian moments.
    $endgroup$
    – zoidberg
    Dec 12 '18 at 2:04










  • $begingroup$
    Actually, take a look at this: math.stackexchange.com/questions/1652781/…
    $endgroup$
    – zoidberg
    Dec 12 '18 at 2:13










  • $begingroup$
    So I think the bound you'll get will be $e^{-Omega(sqrt{t})}$
    $endgroup$
    – zoidberg
    Dec 12 '18 at 2:56










  • $begingroup$
    I think you are right. It seems that $frac{1}{b}sum_{i=1}^kX_i^2Y_i^2>(frac{1}{b}sum_{i=1}^kX_i^2)^2$ close to 50% of the time. So the values seems very similar. Using the later value we get $expleft(frac{-k}{4} left(2 sqrt{t}-2-log (t)right)right)$, which would be great.
    $endgroup$
    – Thomas Ahle
    Dec 12 '18 at 15:56


















$begingroup$
Hm, I'm not sure you can do much better than the Markov inequality bound on high moments. $X$ is a sub-gaussian variable, so $X^2$ is a sub-exponential, which means $X^2Y^2$ is worse than that. As you said, the mgf doesn't exist so your best bet is to find bounds on high moments. It's probably unavoidable to expand things out and wade into the combinatorics of gaussian moments.
$endgroup$
– zoidberg
Dec 12 '18 at 2:04




$begingroup$
Hm, I'm not sure you can do much better than the Markov inequality bound on high moments. $X$ is a sub-gaussian variable, so $X^2$ is a sub-exponential, which means $X^2Y^2$ is worse than that. As you said, the mgf doesn't exist so your best bet is to find bounds on high moments. It's probably unavoidable to expand things out and wade into the combinatorics of gaussian moments.
$endgroup$
– zoidberg
Dec 12 '18 at 2:04












$begingroup$
Actually, take a look at this: math.stackexchange.com/questions/1652781/…
$endgroup$
– zoidberg
Dec 12 '18 at 2:13




$begingroup$
Actually, take a look at this: math.stackexchange.com/questions/1652781/…
$endgroup$
– zoidberg
Dec 12 '18 at 2:13












$begingroup$
So I think the bound you'll get will be $e^{-Omega(sqrt{t})}$
$endgroup$
– zoidberg
Dec 12 '18 at 2:56




$begingroup$
So I think the bound you'll get will be $e^{-Omega(sqrt{t})}$
$endgroup$
– zoidberg
Dec 12 '18 at 2:56












$begingroup$
I think you are right. It seems that $frac{1}{b}sum_{i=1}^kX_i^2Y_i^2>(frac{1}{b}sum_{i=1}^kX_i^2)^2$ close to 50% of the time. So the values seems very similar. Using the later value we get $expleft(frac{-k}{4} left(2 sqrt{t}-2-log (t)right)right)$, which would be great.
$endgroup$
– Thomas Ahle
Dec 12 '18 at 15:56






$begingroup$
I think you are right. It seems that $frac{1}{b}sum_{i=1}^kX_i^2Y_i^2>(frac{1}{b}sum_{i=1}^kX_i^2)^2$ close to 50% of the time. So the values seems very similar. Using the later value we get $expleft(frac{-k}{4} left(2 sqrt{t}-2-log (t)right)right)$, which would be great.
$endgroup$
– Thomas Ahle
Dec 12 '18 at 15:56












1 Answer
1






active

oldest

votes


















1












$begingroup$

I found that a good way to handle this sum is using Bernstein's inequality:




Let $X_1, ldots, X_n$ be independent zero-mean random variables. Suppose that $|X_i|leq M$ almost surely, for all $i$. Then, for all positive
$t$,



$$Pr left (sum_{i=1}^n X_i ge t right ) leq exp left(
-frac{t^2/2}{sum mathbb{E} X^2+ Mt/3} right).$$




We have
$$
Pr[X^2Y^2ge t]
le E[(XY)^{2p}]t^{-p}
le 2(2p/e)^{2p}t^{-p}
le 2 e^{-sqrt t},
$$

taking $p=sqrt{t}/2$. Hence we can take a union bound over all $XY$ to get:
$$
begin{align}
Prleft(
sum_k X_k^2Y_k^2ge (1+epsilon)k
right)
&le
exp left(
-frac{kepsilon^2/2}{9+ Mepsilon/3} right)
+2k exp(-sqrt M)le2delta
end{align}
$$

When taking $M=log^2((2 k)/delta)$ and
$$begin{align}t
&=
frac{18 log left(frac{1}{delta}right)}{epsilon^2}+frac{2 log left(frac{1}{delta}right) log^2 left(frac{2 k}{d}right)}{3 epsilon}
=O(epsilon^{-2}log1/delta+epsilon^{-1}log^32k/delta).
end{align}$$



This matches exactly what we would expect from the central limit theory in the first term, and nearly bound from a "single large term" in the second term.



The only remaining question is whether we can get rid of the third power, to just have $epsilon^{-1}log^2(k/delta)$ in the second term. I don't know how to do that though.



Update



The previous approach lost a factor $log1/delta$ in $t$.
This can be avoided by the following neat trick: Instead of using Bernstein's inequality, just use $(XY)^2le M^{1/2}|XY|le sqrt M (X^2+Y^2)/2$.



That gives us
$$begin{align}
Pr[sum_iX_i^2Y_i^2 ge (1+epsilon)k mid X^2Y^2le M]
&le expleft(lambda k X^2Y^2kright)/expleft(lambda(1+epsilon)right)
\&le expleft(lambda k sqrt M (X^2+Y^2)/2right)/expleft(lambda(1+epsilon)kright)
\&= frac{1}{1-lambda ksqrt M}/expleft(lambda(1+epsilon)kright)
\&=frac{(1+epsilon)k}{sqrt M}exp(1 - frac{(1+epsilon)k}{sqrt M})
\&=frac{(1+epsilon)k}{log(2k/delta)}exp(1 - frac{(1+epsilon)k}{log(2k/delta)})
%\&=sqrt{(1+epsilon)k}exp(1 - sqrt{(1+epsilon)k}).
end{align}$$



Taking $lambda=((1+epsilon)k-M)/(M (1+epsilon)k)$ and $M=(1+epsilon)k$.
The union bound over from the single elements is then $+2kexp(-sqrt{(1+epsilon)k})$.



We see that taking $kapproxepsilon^{-2}log1/delta + epsilon^{-1}(log1/delta)^2$ now suffices.



The method is nearly as versatile as the Bernstein approach, and it gives the optimal values.






share|cite|improve this answer











$endgroup$













    Your Answer





    StackExchange.ifUsing("editor", function () {
    return StackExchange.using("mathjaxEditing", function () {
    StackExchange.MarkdownEditor.creationCallbacks.add(function (editor, postfix) {
    StackExchange.mathjaxEditing.prepareWmdForMathJax(editor, postfix, [["$", "$"], ["\\(","\\)"]]);
    });
    });
    }, "mathjax-editing");

    StackExchange.ready(function() {
    var channelOptions = {
    tags: "".split(" "),
    id: "69"
    };
    initTagRenderer("".split(" "), "".split(" "), channelOptions);

    StackExchange.using("externalEditor", function() {
    // Have to fire editor after snippets, if snippets enabled
    if (StackExchange.settings.snippets.snippetsEnabled) {
    StackExchange.using("snippets", function() {
    createEditor();
    });
    }
    else {
    createEditor();
    }
    });

    function createEditor() {
    StackExchange.prepareEditor({
    heartbeatType: 'answer',
    autoActivateHeartbeat: false,
    convertImagesToLinks: true,
    noModals: true,
    showLowRepImageUploadWarning: true,
    reputationToPostImages: 10,
    bindNavPrevention: true,
    postfix: "",
    imageUploader: {
    brandingHtml: "Powered by u003ca class="icon-imgur-white" href="https://imgur.com/"u003eu003c/au003e",
    contentPolicyHtml: "User contributions licensed under u003ca href="https://creativecommons.org/licenses/by-sa/3.0/"u003ecc by-sa 3.0 with attribution requiredu003c/au003e u003ca href="https://stackoverflow.com/legal/content-policy"u003e(content policy)u003c/au003e",
    allowUrls: true
    },
    noCode: true, onDemand: true,
    discardSelector: ".discard-answer"
    ,immediatelyShowMarkdownHelp:true
    });


    }
    });














    draft saved

    draft discarded


















    StackExchange.ready(
    function () {
    StackExchange.openid.initPostLogin('.new-post-login', 'https%3a%2f%2fmath.stackexchange.com%2fquestions%2f3035194%2fpr-sum-i-x-i2-y-i2-ge-t-chernoff-bound-for-sum-of-pairs-of-squared-norma%23new-answer', 'question_page');
    }
    );

    Post as a guest















    Required, but never shown

























    1 Answer
    1






    active

    oldest

    votes








    1 Answer
    1






    active

    oldest

    votes









    active

    oldest

    votes






    active

    oldest

    votes









    1












    $begingroup$

    I found that a good way to handle this sum is using Bernstein's inequality:




    Let $X_1, ldots, X_n$ be independent zero-mean random variables. Suppose that $|X_i|leq M$ almost surely, for all $i$. Then, for all positive
    $t$,



    $$Pr left (sum_{i=1}^n X_i ge t right ) leq exp left(
    -frac{t^2/2}{sum mathbb{E} X^2+ Mt/3} right).$$




    We have
    $$
    Pr[X^2Y^2ge t]
    le E[(XY)^{2p}]t^{-p}
    le 2(2p/e)^{2p}t^{-p}
    le 2 e^{-sqrt t},
    $$

    taking $p=sqrt{t}/2$. Hence we can take a union bound over all $XY$ to get:
    $$
    begin{align}
    Prleft(
    sum_k X_k^2Y_k^2ge (1+epsilon)k
    right)
    &le
    exp left(
    -frac{kepsilon^2/2}{9+ Mepsilon/3} right)
    +2k exp(-sqrt M)le2delta
    end{align}
    $$

    When taking $M=log^2((2 k)/delta)$ and
    $$begin{align}t
    &=
    frac{18 log left(frac{1}{delta}right)}{epsilon^2}+frac{2 log left(frac{1}{delta}right) log^2 left(frac{2 k}{d}right)}{3 epsilon}
    =O(epsilon^{-2}log1/delta+epsilon^{-1}log^32k/delta).
    end{align}$$



    This matches exactly what we would expect from the central limit theory in the first term, and nearly bound from a "single large term" in the second term.



    The only remaining question is whether we can get rid of the third power, to just have $epsilon^{-1}log^2(k/delta)$ in the second term. I don't know how to do that though.



    Update



    The previous approach lost a factor $log1/delta$ in $t$.
    This can be avoided by the following neat trick: Instead of using Bernstein's inequality, just use $(XY)^2le M^{1/2}|XY|le sqrt M (X^2+Y^2)/2$.



    That gives us
    $$begin{align}
    Pr[sum_iX_i^2Y_i^2 ge (1+epsilon)k mid X^2Y^2le M]
    &le expleft(lambda k X^2Y^2kright)/expleft(lambda(1+epsilon)right)
    \&le expleft(lambda k sqrt M (X^2+Y^2)/2right)/expleft(lambda(1+epsilon)kright)
    \&= frac{1}{1-lambda ksqrt M}/expleft(lambda(1+epsilon)kright)
    \&=frac{(1+epsilon)k}{sqrt M}exp(1 - frac{(1+epsilon)k}{sqrt M})
    \&=frac{(1+epsilon)k}{log(2k/delta)}exp(1 - frac{(1+epsilon)k}{log(2k/delta)})
    %\&=sqrt{(1+epsilon)k}exp(1 - sqrt{(1+epsilon)k}).
    end{align}$$



    Taking $lambda=((1+epsilon)k-M)/(M (1+epsilon)k)$ and $M=(1+epsilon)k$.
    The union bound over from the single elements is then $+2kexp(-sqrt{(1+epsilon)k})$.



    We see that taking $kapproxepsilon^{-2}log1/delta + epsilon^{-1}(log1/delta)^2$ now suffices.



    The method is nearly as versatile as the Bernstein approach, and it gives the optimal values.






    share|cite|improve this answer











    $endgroup$


















      1












      $begingroup$

      I found that a good way to handle this sum is using Bernstein's inequality:




      Let $X_1, ldots, X_n$ be independent zero-mean random variables. Suppose that $|X_i|leq M$ almost surely, for all $i$. Then, for all positive
      $t$,



      $$Pr left (sum_{i=1}^n X_i ge t right ) leq exp left(
      -frac{t^2/2}{sum mathbb{E} X^2+ Mt/3} right).$$




      We have
      $$
      Pr[X^2Y^2ge t]
      le E[(XY)^{2p}]t^{-p}
      le 2(2p/e)^{2p}t^{-p}
      le 2 e^{-sqrt t},
      $$

      taking $p=sqrt{t}/2$. Hence we can take a union bound over all $XY$ to get:
      $$
      begin{align}
      Prleft(
      sum_k X_k^2Y_k^2ge (1+epsilon)k
      right)
      &le
      exp left(
      -frac{kepsilon^2/2}{9+ Mepsilon/3} right)
      +2k exp(-sqrt M)le2delta
      end{align}
      $$

      When taking $M=log^2((2 k)/delta)$ and
      $$begin{align}t
      &=
      frac{18 log left(frac{1}{delta}right)}{epsilon^2}+frac{2 log left(frac{1}{delta}right) log^2 left(frac{2 k}{d}right)}{3 epsilon}
      =O(epsilon^{-2}log1/delta+epsilon^{-1}log^32k/delta).
      end{align}$$



      This matches exactly what we would expect from the central limit theory in the first term, and nearly bound from a "single large term" in the second term.



      The only remaining question is whether we can get rid of the third power, to just have $epsilon^{-1}log^2(k/delta)$ in the second term. I don't know how to do that though.



      Update



      The previous approach lost a factor $log1/delta$ in $t$.
      This can be avoided by the following neat trick: Instead of using Bernstein's inequality, just use $(XY)^2le M^{1/2}|XY|le sqrt M (X^2+Y^2)/2$.



      That gives us
      $$begin{align}
      Pr[sum_iX_i^2Y_i^2 ge (1+epsilon)k mid X^2Y^2le M]
      &le expleft(lambda k X^2Y^2kright)/expleft(lambda(1+epsilon)right)
      \&le expleft(lambda k sqrt M (X^2+Y^2)/2right)/expleft(lambda(1+epsilon)kright)
      \&= frac{1}{1-lambda ksqrt M}/expleft(lambda(1+epsilon)kright)
      \&=frac{(1+epsilon)k}{sqrt M}exp(1 - frac{(1+epsilon)k}{sqrt M})
      \&=frac{(1+epsilon)k}{log(2k/delta)}exp(1 - frac{(1+epsilon)k}{log(2k/delta)})
      %\&=sqrt{(1+epsilon)k}exp(1 - sqrt{(1+epsilon)k}).
      end{align}$$



      Taking $lambda=((1+epsilon)k-M)/(M (1+epsilon)k)$ and $M=(1+epsilon)k$.
      The union bound over from the single elements is then $+2kexp(-sqrt{(1+epsilon)k})$.



      We see that taking $kapproxepsilon^{-2}log1/delta + epsilon^{-1}(log1/delta)^2$ now suffices.



      The method is nearly as versatile as the Bernstein approach, and it gives the optimal values.






      share|cite|improve this answer











      $endgroup$
















        1












        1








        1





        $begingroup$

        I found that a good way to handle this sum is using Bernstein's inequality:




        Let $X_1, ldots, X_n$ be independent zero-mean random variables. Suppose that $|X_i|leq M$ almost surely, for all $i$. Then, for all positive
        $t$,



        $$Pr left (sum_{i=1}^n X_i ge t right ) leq exp left(
        -frac{t^2/2}{sum mathbb{E} X^2+ Mt/3} right).$$




        We have
        $$
        Pr[X^2Y^2ge t]
        le E[(XY)^{2p}]t^{-p}
        le 2(2p/e)^{2p}t^{-p}
        le 2 e^{-sqrt t},
        $$

        taking $p=sqrt{t}/2$. Hence we can take a union bound over all $XY$ to get:
        $$
        begin{align}
        Prleft(
        sum_k X_k^2Y_k^2ge (1+epsilon)k
        right)
        &le
        exp left(
        -frac{kepsilon^2/2}{9+ Mepsilon/3} right)
        +2k exp(-sqrt M)le2delta
        end{align}
        $$

        When taking $M=log^2((2 k)/delta)$ and
        $$begin{align}t
        &=
        frac{18 log left(frac{1}{delta}right)}{epsilon^2}+frac{2 log left(frac{1}{delta}right) log^2 left(frac{2 k}{d}right)}{3 epsilon}
        =O(epsilon^{-2}log1/delta+epsilon^{-1}log^32k/delta).
        end{align}$$



        This matches exactly what we would expect from the central limit theory in the first term, and nearly bound from a "single large term" in the second term.



        The only remaining question is whether we can get rid of the third power, to just have $epsilon^{-1}log^2(k/delta)$ in the second term. I don't know how to do that though.



        Update



        The previous approach lost a factor $log1/delta$ in $t$.
        This can be avoided by the following neat trick: Instead of using Bernstein's inequality, just use $(XY)^2le M^{1/2}|XY|le sqrt M (X^2+Y^2)/2$.



        That gives us
        $$begin{align}
        Pr[sum_iX_i^2Y_i^2 ge (1+epsilon)k mid X^2Y^2le M]
        &le expleft(lambda k X^2Y^2kright)/expleft(lambda(1+epsilon)right)
        \&le expleft(lambda k sqrt M (X^2+Y^2)/2right)/expleft(lambda(1+epsilon)kright)
        \&= frac{1}{1-lambda ksqrt M}/expleft(lambda(1+epsilon)kright)
        \&=frac{(1+epsilon)k}{sqrt M}exp(1 - frac{(1+epsilon)k}{sqrt M})
        \&=frac{(1+epsilon)k}{log(2k/delta)}exp(1 - frac{(1+epsilon)k}{log(2k/delta)})
        %\&=sqrt{(1+epsilon)k}exp(1 - sqrt{(1+epsilon)k}).
        end{align}$$



        Taking $lambda=((1+epsilon)k-M)/(M (1+epsilon)k)$ and $M=(1+epsilon)k$.
        The union bound over from the single elements is then $+2kexp(-sqrt{(1+epsilon)k})$.



        We see that taking $kapproxepsilon^{-2}log1/delta + epsilon^{-1}(log1/delta)^2$ now suffices.



        The method is nearly as versatile as the Bernstein approach, and it gives the optimal values.






        share|cite|improve this answer











        $endgroup$



        I found that a good way to handle this sum is using Bernstein's inequality:




        Let $X_1, ldots, X_n$ be independent zero-mean random variables. Suppose that $|X_i|leq M$ almost surely, for all $i$. Then, for all positive
        $t$,



        $$Pr left (sum_{i=1}^n X_i ge t right ) leq exp left(
        -frac{t^2/2}{sum mathbb{E} X^2+ Mt/3} right).$$




        We have
        $$
        Pr[X^2Y^2ge t]
        le E[(XY)^{2p}]t^{-p}
        le 2(2p/e)^{2p}t^{-p}
        le 2 e^{-sqrt t},
        $$

        taking $p=sqrt{t}/2$. Hence we can take a union bound over all $XY$ to get:
        $$
        begin{align}
        Prleft(
        sum_k X_k^2Y_k^2ge (1+epsilon)k
        right)
        &le
        exp left(
        -frac{kepsilon^2/2}{9+ Mepsilon/3} right)
        +2k exp(-sqrt M)le2delta
        end{align}
        $$

        When taking $M=log^2((2 k)/delta)$ and
        $$begin{align}t
        &=
        frac{18 log left(frac{1}{delta}right)}{epsilon^2}+frac{2 log left(frac{1}{delta}right) log^2 left(frac{2 k}{d}right)}{3 epsilon}
        =O(epsilon^{-2}log1/delta+epsilon^{-1}log^32k/delta).
        end{align}$$



        This matches exactly what we would expect from the central limit theory in the first term, and nearly bound from a "single large term" in the second term.



        The only remaining question is whether we can get rid of the third power, to just have $epsilon^{-1}log^2(k/delta)$ in the second term. I don't know how to do that though.



        Update



        The previous approach lost a factor $log1/delta$ in $t$.
        This can be avoided by the following neat trick: Instead of using Bernstein's inequality, just use $(XY)^2le M^{1/2}|XY|le sqrt M (X^2+Y^2)/2$.



        That gives us
        $$begin{align}
        Pr[sum_iX_i^2Y_i^2 ge (1+epsilon)k mid X^2Y^2le M]
        &le expleft(lambda k X^2Y^2kright)/expleft(lambda(1+epsilon)right)
        \&le expleft(lambda k sqrt M (X^2+Y^2)/2right)/expleft(lambda(1+epsilon)kright)
        \&= frac{1}{1-lambda ksqrt M}/expleft(lambda(1+epsilon)kright)
        \&=frac{(1+epsilon)k}{sqrt M}exp(1 - frac{(1+epsilon)k}{sqrt M})
        \&=frac{(1+epsilon)k}{log(2k/delta)}exp(1 - frac{(1+epsilon)k}{log(2k/delta)})
        %\&=sqrt{(1+epsilon)k}exp(1 - sqrt{(1+epsilon)k}).
        end{align}$$



        Taking $lambda=((1+epsilon)k-M)/(M (1+epsilon)k)$ and $M=(1+epsilon)k$.
        The union bound over from the single elements is then $+2kexp(-sqrt{(1+epsilon)k})$.



        We see that taking $kapproxepsilon^{-2}log1/delta + epsilon^{-1}(log1/delta)^2$ now suffices.



        The method is nearly as versatile as the Bernstein approach, and it gives the optimal values.







        share|cite|improve this answer














        share|cite|improve this answer



        share|cite|improve this answer








        edited Jan 11 at 16:47

























        answered Dec 21 '18 at 18:57









        Thomas AhleThomas Ahle

        1,5121320




        1,5121320






























            draft saved

            draft discarded




















































            Thanks for contributing an answer to Mathematics Stack Exchange!


            • Please be sure to answer the question. Provide details and share your research!

            But avoid



            • Asking for help, clarification, or responding to other answers.

            • Making statements based on opinion; back them up with references or personal experience.


            Use MathJax to format equations. MathJax reference.


            To learn more, see our tips on writing great answers.




            draft saved


            draft discarded














            StackExchange.ready(
            function () {
            StackExchange.openid.initPostLogin('.new-post-login', 'https%3a%2f%2fmath.stackexchange.com%2fquestions%2f3035194%2fpr-sum-i-x-i2-y-i2-ge-t-chernoff-bound-for-sum-of-pairs-of-squared-norma%23new-answer', 'question_page');
            }
            );

            Post as a guest















            Required, but never shown





















































            Required, but never shown














            Required, but never shown












            Required, but never shown







            Required, but never shown

































            Required, but never shown














            Required, but never shown












            Required, but never shown







            Required, but never shown







            Popular posts from this blog

            Bundesstraße 106

            Ida-Boy-Ed-Garten

            Le Mesnil-Réaume